Integral from infinity to infinityWhat is Leibnitz rule for a double integral?Can a substitution cause a convergent definite integral to diverge?Weird computation error when using fnInt (numerical integral) on TI-84 PlusIs there a change-of-variables solution for integrals from negative infinity to a constant?What's exactly the deal with differentials? (Confessions of a desperate calculus student)Quadrature integration: calculating the weightsDoes this improper integral converge or is it a trick question?Check whether $p(x,y) = xye^-x-y$ is a joint density functionImproper Integral Convergence involving $e^x$Weird integrals

I hit a pipe with a mower and now it won't turn

Who voices the character "Finger" in The Fifth Element?

Boolean Difference with Offset?

Is this homebrew Half-Phoenix race balanced?

Heuristics for mixed integer linear and nonlinear programs

Why do we use a cylinder as a Gaussian surface for infinitely long charged wire?

Can an editor review manuscript without sending to reviewers?

What is the purpose of putting a capacitor on the primary side of a step-down transformer?

Find first and last non-zero column in each row of a pandas dataframe

What does grep -v "grep" mean and do?

How did researchers find articles before the Internet and the computer era?

Using a concentration spell on top of another spell from another spell list?

What game is this character in the Pixels movie from?

What's the easiest way for a whole party to be able to communicate with a creature that doesn't know Common?

Could the Q destroy the universe?

Is there reliable evidence that depleted uranium from the 1999 NATO bombing is causing cancer in Serbia?

If two black hole event horizons overlap (touch) can they ever separate again?

What exactly did Ant-Man see that made him say that their plan worked?

Is the location of an aircraft spoiler really that vital?

Handling a player (unintentionally) stealing the spotlight

How to securely dispose of a smartphone?

One folder having two different locations on Ubuntu 18.04

Pairwise Scatter Plots with Histograms and Correlations

Why would anyone even use a Portkey?



Integral from infinity to infinity


What is Leibnitz rule for a double integral?Can a substitution cause a convergent definite integral to diverge?Weird computation error when using fnInt (numerical integral) on TI-84 PlusIs there a change-of-variables solution for integrals from negative infinity to a constant?What's exactly the deal with differentials? (Confessions of a desperate calculus student)Quadrature integration: calculating the weightsDoes this improper integral converge or is it a trick question?Check whether $p(x,y) = xye^-x-y$ is a joint density functionImproper Integral Convergence involving $e^x$Weird integrals






.everyoneloves__top-leaderboard:empty,.everyoneloves__mid-leaderboard:empty,.everyoneloves__bot-mid-leaderboard:empty margin-bottom:0;








4












$begingroup$


My physics professor today wrote on the blackboard:
$$ int_infty^infty f(x) dx = 0 $$
for every function $f$.
And the proof he gave was:
$$ int_infty^infty f(x) dx = int_infty^a f(x) dx + int_a^infty f(x)dx = - int_a^infty f(x) dx + int_a^inftyf(x)dx = 0$$



However I'm still not convinced, for me an integral from infinity to infinity has no meaning. Therefore, what I'm asking is: does the above equations make sense? If not, are there cases where they do make sense? I'm thinking about functions that converge to 0 in $+infty$.



EDIT: Actually, the function f considered was a density, i.e.:
$$ int_-infty^+infty f(x)dx = 1 $$
and $f(x) geq 0$ for all $x$.










share|cite|improve this question











$endgroup$







  • 4




    $begingroup$
    Not much IMHO. $$
    $endgroup$
    – José Carlos Santos
    8 hours ago







  • 2




    $begingroup$
    I hope your professor meant this as a joke. Did he (or she) actually go on to use this result in an argument?
    $endgroup$
    – Ethan Bolker
    8 hours ago










  • $begingroup$
    How can the integral be 0 and 1 at the same time?
    $endgroup$
    – copper.hat
    8 hours ago










  • $begingroup$
    @copper.hat one integral is from -infinity to infinity, the other one is from +infinity to +infinity
    $endgroup$
    – Victor
    8 hours ago










  • $begingroup$
    I see. It is a bit meaningless. I would not spend too much time pondering it. It is like $int_a^a$ where $a$ is finite.
    $endgroup$
    – copper.hat
    7 hours ago

















4












$begingroup$


My physics professor today wrote on the blackboard:
$$ int_infty^infty f(x) dx = 0 $$
for every function $f$.
And the proof he gave was:
$$ int_infty^infty f(x) dx = int_infty^a f(x) dx + int_a^infty f(x)dx = - int_a^infty f(x) dx + int_a^inftyf(x)dx = 0$$



However I'm still not convinced, for me an integral from infinity to infinity has no meaning. Therefore, what I'm asking is: does the above equations make sense? If not, are there cases where they do make sense? I'm thinking about functions that converge to 0 in $+infty$.



EDIT: Actually, the function f considered was a density, i.e.:
$$ int_-infty^+infty f(x)dx = 1 $$
and $f(x) geq 0$ for all $x$.










share|cite|improve this question











$endgroup$







  • 4




    $begingroup$
    Not much IMHO. $$
    $endgroup$
    – José Carlos Santos
    8 hours ago







  • 2




    $begingroup$
    I hope your professor meant this as a joke. Did he (or she) actually go on to use this result in an argument?
    $endgroup$
    – Ethan Bolker
    8 hours ago










  • $begingroup$
    How can the integral be 0 and 1 at the same time?
    $endgroup$
    – copper.hat
    8 hours ago










  • $begingroup$
    @copper.hat one integral is from -infinity to infinity, the other one is from +infinity to +infinity
    $endgroup$
    – Victor
    8 hours ago










  • $begingroup$
    I see. It is a bit meaningless. I would not spend too much time pondering it. It is like $int_a^a$ where $a$ is finite.
    $endgroup$
    – copper.hat
    7 hours ago













4












4








4





$begingroup$


My physics professor today wrote on the blackboard:
$$ int_infty^infty f(x) dx = 0 $$
for every function $f$.
And the proof he gave was:
$$ int_infty^infty f(x) dx = int_infty^a f(x) dx + int_a^infty f(x)dx = - int_a^infty f(x) dx + int_a^inftyf(x)dx = 0$$



However I'm still not convinced, for me an integral from infinity to infinity has no meaning. Therefore, what I'm asking is: does the above equations make sense? If not, are there cases where they do make sense? I'm thinking about functions that converge to 0 in $+infty$.



EDIT: Actually, the function f considered was a density, i.e.:
$$ int_-infty^+infty f(x)dx = 1 $$
and $f(x) geq 0$ for all $x$.










share|cite|improve this question











$endgroup$




My physics professor today wrote on the blackboard:
$$ int_infty^infty f(x) dx = 0 $$
for every function $f$.
And the proof he gave was:
$$ int_infty^infty f(x) dx = int_infty^a f(x) dx + int_a^infty f(x)dx = - int_a^infty f(x) dx + int_a^inftyf(x)dx = 0$$



However I'm still not convinced, for me an integral from infinity to infinity has no meaning. Therefore, what I'm asking is: does the above equations make sense? If not, are there cases where they do make sense? I'm thinking about functions that converge to 0 in $+infty$.



EDIT: Actually, the function f considered was a density, i.e.:
$$ int_-infty^+infty f(x)dx = 1 $$
and $f(x) geq 0$ for all $x$.







integration






share|cite|improve this question















share|cite|improve this question













share|cite|improve this question




share|cite|improve this question








edited 5 hours ago







Victor

















asked 8 hours ago









VictorVictor

777 bronze badges




777 bronze badges







  • 4




    $begingroup$
    Not much IMHO. $$
    $endgroup$
    – José Carlos Santos
    8 hours ago







  • 2




    $begingroup$
    I hope your professor meant this as a joke. Did he (or she) actually go on to use this result in an argument?
    $endgroup$
    – Ethan Bolker
    8 hours ago










  • $begingroup$
    How can the integral be 0 and 1 at the same time?
    $endgroup$
    – copper.hat
    8 hours ago










  • $begingroup$
    @copper.hat one integral is from -infinity to infinity, the other one is from +infinity to +infinity
    $endgroup$
    – Victor
    8 hours ago










  • $begingroup$
    I see. It is a bit meaningless. I would not spend too much time pondering it. It is like $int_a^a$ where $a$ is finite.
    $endgroup$
    – copper.hat
    7 hours ago












  • 4




    $begingroup$
    Not much IMHO. $$
    $endgroup$
    – José Carlos Santos
    8 hours ago







  • 2




    $begingroup$
    I hope your professor meant this as a joke. Did he (or she) actually go on to use this result in an argument?
    $endgroup$
    – Ethan Bolker
    8 hours ago










  • $begingroup$
    How can the integral be 0 and 1 at the same time?
    $endgroup$
    – copper.hat
    8 hours ago










  • $begingroup$
    @copper.hat one integral is from -infinity to infinity, the other one is from +infinity to +infinity
    $endgroup$
    – Victor
    8 hours ago










  • $begingroup$
    I see. It is a bit meaningless. I would not spend too much time pondering it. It is like $int_a^a$ where $a$ is finite.
    $endgroup$
    – copper.hat
    7 hours ago







4




4




$begingroup$
Not much IMHO. $$
$endgroup$
– José Carlos Santos
8 hours ago





$begingroup$
Not much IMHO. $$
$endgroup$
– José Carlos Santos
8 hours ago





2




2




$begingroup$
I hope your professor meant this as a joke. Did he (or she) actually go on to use this result in an argument?
$endgroup$
– Ethan Bolker
8 hours ago




$begingroup$
I hope your professor meant this as a joke. Did he (or she) actually go on to use this result in an argument?
$endgroup$
– Ethan Bolker
8 hours ago












$begingroup$
How can the integral be 0 and 1 at the same time?
$endgroup$
– copper.hat
8 hours ago




$begingroup$
How can the integral be 0 and 1 at the same time?
$endgroup$
– copper.hat
8 hours ago












$begingroup$
@copper.hat one integral is from -infinity to infinity, the other one is from +infinity to +infinity
$endgroup$
– Victor
8 hours ago




$begingroup$
@copper.hat one integral is from -infinity to infinity, the other one is from +infinity to +infinity
$endgroup$
– Victor
8 hours ago












$begingroup$
I see. It is a bit meaningless. I would not spend too much time pondering it. It is like $int_a^a$ where $a$ is finite.
$endgroup$
– copper.hat
7 hours ago




$begingroup$
I see. It is a bit meaningless. I would not spend too much time pondering it. It is like $int_a^a$ where $a$ is finite.
$endgroup$
– copper.hat
7 hours ago










4 Answers
4






active

oldest

votes


















12












$begingroup$

This is not necessarily true. Take the following example;
$$int_a^2afrac1xmathrmdx=[ln]_a^2a=ln(2)$$
If we take $atoinfty$ then the integral becomes
$$int_infty^inftyfrac1xmathrmdx=ln(2)$$
as the integral is constant for all $ainmathbbR$. What I guess your professor meant was that
$$lim_atoinftyint_a^a f(x)mathrmdx=0$$
which is trivially true as the LHS is constantly zero.






share|cite|improve this answer









$endgroup$




















    4












    $begingroup$

    An improper integral with an endpoint of $infty$ means a limit of proper integrals where the endpoint approaches $infty$. Thus a reasonable definition of
    $int_infty^infty f(x); dx$ would be
    $$ int_infty^infty f(x); dx = lim_a, b to infty int_a^b f(x); dx $$
    This is $0$ if and only if $int_a^infty f(x); dx$ converges for some $a$.



    EDIT: If the double limit is $0$, there is $N$ such that
    $left|int_a^b f(x); dxright| < 1$ for all $N < a < b$.
    For any $epsilon > 0$ there is $M > N$ such that for $b, c > M$,
    $$ left|int_b^c f(x); dx right| = left| int_a^c f(x); dx - int_a^b f(x); dx right|< epsilon$$

    and this implies that $lim_b to infty int_a^b f(x); dx$ exists, i.e.
    $int_a^infty f(x); dx$ converges.



    Conversely, if $int_a^infty f(x); dx = L$ converges, then for any $epsilon > 0$ there is $N$ such that $left|int_a^b f(x); dx - Lright| < epsilon/2$ whenever $b > N$. Then if $b > N$ and $c > N$,
    $$ left| int_b^c f(x); dxright| = left|int_a^c f(x); dx - int_a^b f(x); dx right| < epsilon $$






    share|cite|improve this answer











    $endgroup$












    • $begingroup$
      could you provide more details on the condition $int_a^infty f(x)dx$ converges for some $a$ is a IFF condition
      $endgroup$
      – Victor
      5 hours ago











    • $begingroup$
      How would you define $a,btoinfty$ because as I have shown, a suitably chosen $b(=2a)$ causes the integral to remain non-zero.
      $endgroup$
      – Peter Foreman
      5 hours ago











    • $begingroup$
      @PeterForeman In the usual way: the limit is $L$ if for every $epsilon > 0$ there is $N$ such that for all $a>N$ and $b > N$, $left| int_a^b f(x); dx - L right| < epsilon$. In your example the limit does not exist.
      $endgroup$
      – Robert Israel
      2 hours ago


















    1












    $begingroup$

    As has been pointed out by other answers, this is not always true because the symbol $infty$ can hide many things, even if we work with the extended reals. The actual meaning of the $infty$ is a limiting process as a certain variable becomes arbitrarily large. The upper and lower limits in the $$int_a^b f(x)mathrm d x$$ however may approach $infty$ at different rates, and this is the key point. Your professor's argument works only when $a$ and $b$ approach infinity equally fast, or in other words, when they are of equal order at infinity. In particular it is always valid if $a=b.$



    So what about your professor's "proof"? Well, the ambiguity should now be obvious -- he uses the same symbol $infty$ for things that may behave differently. I think it's safe to assume he's thinking only of the variables are of equal order at infinity. Otherwise his proof breaks down since $infty-infty$ can be anything otherwise.



    PS. However, you say an integral from $infty$ to $infty$ has no meaning to you. Well, I see you're thinking of ordering of the reals here. But note that we're not just dealing with the reals here, but the extended reals. As explained above, the best way to think of it is to think of the limits of the integral as approaching infinity at (not necessarily equal rates). Then it's easy to make sense of it. Another way may be to think of the one-point compactification of the real axis.






    share|cite|improve this answer











    $endgroup$




















      0












      $begingroup$

      As Peter Foreman mentioned, there are some occasions where you will get an integral of this form, however the exact integral does matter. Firstly visualise the error function:
      $$operatornameerf(x)=frac2sqrtpiint_0^xe^-t^2dt$$
      notice that:
      $$lim_xto 0operatornameerf(x)=frac2sqrtpiint_0^0e^-t^2dt$$
      Normally we can visualise an integral as area, but in this form it does not make sense since the range over which the area found $to0$. The most obvious case when this integral is zero, no matter the function, would be when both the bounds are equal as the range of the integral is then zero. Overall, it comes down to notation and making sure the bounds are properly defined.






      share|cite|improve this answer









      $endgroup$















        Your Answer








        StackExchange.ready(function()
        var channelOptions =
        tags: "".split(" "),
        id: "69"
        ;
        initTagRenderer("".split(" "), "".split(" "), channelOptions);

        StackExchange.using("externalEditor", function()
        // Have to fire editor after snippets, if snippets enabled
        if (StackExchange.settings.snippets.snippetsEnabled)
        StackExchange.using("snippets", function()
        createEditor();
        );

        else
        createEditor();

        );

        function createEditor()
        StackExchange.prepareEditor(
        heartbeatType: 'answer',
        autoActivateHeartbeat: false,
        convertImagesToLinks: true,
        noModals: true,
        showLowRepImageUploadWarning: true,
        reputationToPostImages: 10,
        bindNavPrevention: true,
        postfix: "",
        imageUploader:
        brandingHtml: "Powered by u003ca class="icon-imgur-white" href="https://imgur.com/"u003eu003c/au003e",
        contentPolicyHtml: "User contributions licensed under u003ca href="https://creativecommons.org/licenses/by-sa/3.0/"u003ecc by-sa 3.0 with attribution requiredu003c/au003e u003ca href="https://stackoverflow.com/legal/content-policy"u003e(content policy)u003c/au003e",
        allowUrls: true
        ,
        noCode: true, onDemand: true,
        discardSelector: ".discard-answer"
        ,immediatelyShowMarkdownHelp:true
        );



        );













        draft saved

        draft discarded


















        StackExchange.ready(
        function ()
        StackExchange.openid.initPostLogin('.new-post-login', 'https%3a%2f%2fmath.stackexchange.com%2fquestions%2f3274013%2fintegral-from-infinity-to-infinity%23new-answer', 'question_page');

        );

        Post as a guest















        Required, but never shown

























        4 Answers
        4






        active

        oldest

        votes








        4 Answers
        4






        active

        oldest

        votes









        active

        oldest

        votes






        active

        oldest

        votes









        12












        $begingroup$

        This is not necessarily true. Take the following example;
        $$int_a^2afrac1xmathrmdx=[ln]_a^2a=ln(2)$$
        If we take $atoinfty$ then the integral becomes
        $$int_infty^inftyfrac1xmathrmdx=ln(2)$$
        as the integral is constant for all $ainmathbbR$. What I guess your professor meant was that
        $$lim_atoinftyint_a^a f(x)mathrmdx=0$$
        which is trivially true as the LHS is constantly zero.






        share|cite|improve this answer









        $endgroup$

















          12












          $begingroup$

          This is not necessarily true. Take the following example;
          $$int_a^2afrac1xmathrmdx=[ln]_a^2a=ln(2)$$
          If we take $atoinfty$ then the integral becomes
          $$int_infty^inftyfrac1xmathrmdx=ln(2)$$
          as the integral is constant for all $ainmathbbR$. What I guess your professor meant was that
          $$lim_atoinftyint_a^a f(x)mathrmdx=0$$
          which is trivially true as the LHS is constantly zero.






          share|cite|improve this answer









          $endgroup$















            12












            12








            12





            $begingroup$

            This is not necessarily true. Take the following example;
            $$int_a^2afrac1xmathrmdx=[ln]_a^2a=ln(2)$$
            If we take $atoinfty$ then the integral becomes
            $$int_infty^inftyfrac1xmathrmdx=ln(2)$$
            as the integral is constant for all $ainmathbbR$. What I guess your professor meant was that
            $$lim_atoinftyint_a^a f(x)mathrmdx=0$$
            which is trivially true as the LHS is constantly zero.






            share|cite|improve this answer









            $endgroup$



            This is not necessarily true. Take the following example;
            $$int_a^2afrac1xmathrmdx=[ln]_a^2a=ln(2)$$
            If we take $atoinfty$ then the integral becomes
            $$int_infty^inftyfrac1xmathrmdx=ln(2)$$
            as the integral is constant for all $ainmathbbR$. What I guess your professor meant was that
            $$lim_atoinftyint_a^a f(x)mathrmdx=0$$
            which is trivially true as the LHS is constantly zero.







            share|cite|improve this answer












            share|cite|improve this answer



            share|cite|improve this answer










            answered 8 hours ago









            Peter ForemanPeter Foreman

            11k1 gold badge4 silver badges26 bronze badges




            11k1 gold badge4 silver badges26 bronze badges























                4












                $begingroup$

                An improper integral with an endpoint of $infty$ means a limit of proper integrals where the endpoint approaches $infty$. Thus a reasonable definition of
                $int_infty^infty f(x); dx$ would be
                $$ int_infty^infty f(x); dx = lim_a, b to infty int_a^b f(x); dx $$
                This is $0$ if and only if $int_a^infty f(x); dx$ converges for some $a$.



                EDIT: If the double limit is $0$, there is $N$ such that
                $left|int_a^b f(x); dxright| < 1$ for all $N < a < b$.
                For any $epsilon > 0$ there is $M > N$ such that for $b, c > M$,
                $$ left|int_b^c f(x); dx right| = left| int_a^c f(x); dx - int_a^b f(x); dx right|< epsilon$$

                and this implies that $lim_b to infty int_a^b f(x); dx$ exists, i.e.
                $int_a^infty f(x); dx$ converges.



                Conversely, if $int_a^infty f(x); dx = L$ converges, then for any $epsilon > 0$ there is $N$ such that $left|int_a^b f(x); dx - Lright| < epsilon/2$ whenever $b > N$. Then if $b > N$ and $c > N$,
                $$ left| int_b^c f(x); dxright| = left|int_a^c f(x); dx - int_a^b f(x); dx right| < epsilon $$






                share|cite|improve this answer











                $endgroup$












                • $begingroup$
                  could you provide more details on the condition $int_a^infty f(x)dx$ converges for some $a$ is a IFF condition
                  $endgroup$
                  – Victor
                  5 hours ago











                • $begingroup$
                  How would you define $a,btoinfty$ because as I have shown, a suitably chosen $b(=2a)$ causes the integral to remain non-zero.
                  $endgroup$
                  – Peter Foreman
                  5 hours ago











                • $begingroup$
                  @PeterForeman In the usual way: the limit is $L$ if for every $epsilon > 0$ there is $N$ such that for all $a>N$ and $b > N$, $left| int_a^b f(x); dx - L right| < epsilon$. In your example the limit does not exist.
                  $endgroup$
                  – Robert Israel
                  2 hours ago















                4












                $begingroup$

                An improper integral with an endpoint of $infty$ means a limit of proper integrals where the endpoint approaches $infty$. Thus a reasonable definition of
                $int_infty^infty f(x); dx$ would be
                $$ int_infty^infty f(x); dx = lim_a, b to infty int_a^b f(x); dx $$
                This is $0$ if and only if $int_a^infty f(x); dx$ converges for some $a$.



                EDIT: If the double limit is $0$, there is $N$ such that
                $left|int_a^b f(x); dxright| < 1$ for all $N < a < b$.
                For any $epsilon > 0$ there is $M > N$ such that for $b, c > M$,
                $$ left|int_b^c f(x); dx right| = left| int_a^c f(x); dx - int_a^b f(x); dx right|< epsilon$$

                and this implies that $lim_b to infty int_a^b f(x); dx$ exists, i.e.
                $int_a^infty f(x); dx$ converges.



                Conversely, if $int_a^infty f(x); dx = L$ converges, then for any $epsilon > 0$ there is $N$ such that $left|int_a^b f(x); dx - Lright| < epsilon/2$ whenever $b > N$. Then if $b > N$ and $c > N$,
                $$ left| int_b^c f(x); dxright| = left|int_a^c f(x); dx - int_a^b f(x); dx right| < epsilon $$






                share|cite|improve this answer











                $endgroup$












                • $begingroup$
                  could you provide more details on the condition $int_a^infty f(x)dx$ converges for some $a$ is a IFF condition
                  $endgroup$
                  – Victor
                  5 hours ago











                • $begingroup$
                  How would you define $a,btoinfty$ because as I have shown, a suitably chosen $b(=2a)$ causes the integral to remain non-zero.
                  $endgroup$
                  – Peter Foreman
                  5 hours ago











                • $begingroup$
                  @PeterForeman In the usual way: the limit is $L$ if for every $epsilon > 0$ there is $N$ such that for all $a>N$ and $b > N$, $left| int_a^b f(x); dx - L right| < epsilon$. In your example the limit does not exist.
                  $endgroup$
                  – Robert Israel
                  2 hours ago













                4












                4








                4





                $begingroup$

                An improper integral with an endpoint of $infty$ means a limit of proper integrals where the endpoint approaches $infty$. Thus a reasonable definition of
                $int_infty^infty f(x); dx$ would be
                $$ int_infty^infty f(x); dx = lim_a, b to infty int_a^b f(x); dx $$
                This is $0$ if and only if $int_a^infty f(x); dx$ converges for some $a$.



                EDIT: If the double limit is $0$, there is $N$ such that
                $left|int_a^b f(x); dxright| < 1$ for all $N < a < b$.
                For any $epsilon > 0$ there is $M > N$ such that for $b, c > M$,
                $$ left|int_b^c f(x); dx right| = left| int_a^c f(x); dx - int_a^b f(x); dx right|< epsilon$$

                and this implies that $lim_b to infty int_a^b f(x); dx$ exists, i.e.
                $int_a^infty f(x); dx$ converges.



                Conversely, if $int_a^infty f(x); dx = L$ converges, then for any $epsilon > 0$ there is $N$ such that $left|int_a^b f(x); dx - Lright| < epsilon/2$ whenever $b > N$. Then if $b > N$ and $c > N$,
                $$ left| int_b^c f(x); dxright| = left|int_a^c f(x); dx - int_a^b f(x); dx right| < epsilon $$






                share|cite|improve this answer











                $endgroup$



                An improper integral with an endpoint of $infty$ means a limit of proper integrals where the endpoint approaches $infty$. Thus a reasonable definition of
                $int_infty^infty f(x); dx$ would be
                $$ int_infty^infty f(x); dx = lim_a, b to infty int_a^b f(x); dx $$
                This is $0$ if and only if $int_a^infty f(x); dx$ converges for some $a$.



                EDIT: If the double limit is $0$, there is $N$ such that
                $left|int_a^b f(x); dxright| < 1$ for all $N < a < b$.
                For any $epsilon > 0$ there is $M > N$ such that for $b, c > M$,
                $$ left|int_b^c f(x); dx right| = left| int_a^c f(x); dx - int_a^b f(x); dx right|< epsilon$$

                and this implies that $lim_b to infty int_a^b f(x); dx$ exists, i.e.
                $int_a^infty f(x); dx$ converges.



                Conversely, if $int_a^infty f(x); dx = L$ converges, then for any $epsilon > 0$ there is $N$ such that $left|int_a^b f(x); dx - Lright| < epsilon/2$ whenever $b > N$. Then if $b > N$ and $c > N$,
                $$ left| int_b^c f(x); dxright| = left|int_a^c f(x); dx - int_a^b f(x); dx right| < epsilon $$







                share|cite|improve this answer














                share|cite|improve this answer



                share|cite|improve this answer








                edited 2 hours ago

























                answered 8 hours ago









                Robert IsraelRobert Israel

                340k23 gold badges233 silver badges492 bronze badges




                340k23 gold badges233 silver badges492 bronze badges











                • $begingroup$
                  could you provide more details on the condition $int_a^infty f(x)dx$ converges for some $a$ is a IFF condition
                  $endgroup$
                  – Victor
                  5 hours ago











                • $begingroup$
                  How would you define $a,btoinfty$ because as I have shown, a suitably chosen $b(=2a)$ causes the integral to remain non-zero.
                  $endgroup$
                  – Peter Foreman
                  5 hours ago











                • $begingroup$
                  @PeterForeman In the usual way: the limit is $L$ if for every $epsilon > 0$ there is $N$ such that for all $a>N$ and $b > N$, $left| int_a^b f(x); dx - L right| < epsilon$. In your example the limit does not exist.
                  $endgroup$
                  – Robert Israel
                  2 hours ago
















                • $begingroup$
                  could you provide more details on the condition $int_a^infty f(x)dx$ converges for some $a$ is a IFF condition
                  $endgroup$
                  – Victor
                  5 hours ago











                • $begingroup$
                  How would you define $a,btoinfty$ because as I have shown, a suitably chosen $b(=2a)$ causes the integral to remain non-zero.
                  $endgroup$
                  – Peter Foreman
                  5 hours ago











                • $begingroup$
                  @PeterForeman In the usual way: the limit is $L$ if for every $epsilon > 0$ there is $N$ such that for all $a>N$ and $b > N$, $left| int_a^b f(x); dx - L right| < epsilon$. In your example the limit does not exist.
                  $endgroup$
                  – Robert Israel
                  2 hours ago















                $begingroup$
                could you provide more details on the condition $int_a^infty f(x)dx$ converges for some $a$ is a IFF condition
                $endgroup$
                – Victor
                5 hours ago





                $begingroup$
                could you provide more details on the condition $int_a^infty f(x)dx$ converges for some $a$ is a IFF condition
                $endgroup$
                – Victor
                5 hours ago













                $begingroup$
                How would you define $a,btoinfty$ because as I have shown, a suitably chosen $b(=2a)$ causes the integral to remain non-zero.
                $endgroup$
                – Peter Foreman
                5 hours ago





                $begingroup$
                How would you define $a,btoinfty$ because as I have shown, a suitably chosen $b(=2a)$ causes the integral to remain non-zero.
                $endgroup$
                – Peter Foreman
                5 hours ago













                $begingroup$
                @PeterForeman In the usual way: the limit is $L$ if for every $epsilon > 0$ there is $N$ such that for all $a>N$ and $b > N$, $left| int_a^b f(x); dx - L right| < epsilon$. In your example the limit does not exist.
                $endgroup$
                – Robert Israel
                2 hours ago




                $begingroup$
                @PeterForeman In the usual way: the limit is $L$ if for every $epsilon > 0$ there is $N$ such that for all $a>N$ and $b > N$, $left| int_a^b f(x); dx - L right| < epsilon$. In your example the limit does not exist.
                $endgroup$
                – Robert Israel
                2 hours ago











                1












                $begingroup$

                As has been pointed out by other answers, this is not always true because the symbol $infty$ can hide many things, even if we work with the extended reals. The actual meaning of the $infty$ is a limiting process as a certain variable becomes arbitrarily large. The upper and lower limits in the $$int_a^b f(x)mathrm d x$$ however may approach $infty$ at different rates, and this is the key point. Your professor's argument works only when $a$ and $b$ approach infinity equally fast, or in other words, when they are of equal order at infinity. In particular it is always valid if $a=b.$



                So what about your professor's "proof"? Well, the ambiguity should now be obvious -- he uses the same symbol $infty$ for things that may behave differently. I think it's safe to assume he's thinking only of the variables are of equal order at infinity. Otherwise his proof breaks down since $infty-infty$ can be anything otherwise.



                PS. However, you say an integral from $infty$ to $infty$ has no meaning to you. Well, I see you're thinking of ordering of the reals here. But note that we're not just dealing with the reals here, but the extended reals. As explained above, the best way to think of it is to think of the limits of the integral as approaching infinity at (not necessarily equal rates). Then it's easy to make sense of it. Another way may be to think of the one-point compactification of the real axis.






                share|cite|improve this answer











                $endgroup$

















                  1












                  $begingroup$

                  As has been pointed out by other answers, this is not always true because the symbol $infty$ can hide many things, even if we work with the extended reals. The actual meaning of the $infty$ is a limiting process as a certain variable becomes arbitrarily large. The upper and lower limits in the $$int_a^b f(x)mathrm d x$$ however may approach $infty$ at different rates, and this is the key point. Your professor's argument works only when $a$ and $b$ approach infinity equally fast, or in other words, when they are of equal order at infinity. In particular it is always valid if $a=b.$



                  So what about your professor's "proof"? Well, the ambiguity should now be obvious -- he uses the same symbol $infty$ for things that may behave differently. I think it's safe to assume he's thinking only of the variables are of equal order at infinity. Otherwise his proof breaks down since $infty-infty$ can be anything otherwise.



                  PS. However, you say an integral from $infty$ to $infty$ has no meaning to you. Well, I see you're thinking of ordering of the reals here. But note that we're not just dealing with the reals here, but the extended reals. As explained above, the best way to think of it is to think of the limits of the integral as approaching infinity at (not necessarily equal rates). Then it's easy to make sense of it. Another way may be to think of the one-point compactification of the real axis.






                  share|cite|improve this answer











                  $endgroup$















                    1












                    1








                    1





                    $begingroup$

                    As has been pointed out by other answers, this is not always true because the symbol $infty$ can hide many things, even if we work with the extended reals. The actual meaning of the $infty$ is a limiting process as a certain variable becomes arbitrarily large. The upper and lower limits in the $$int_a^b f(x)mathrm d x$$ however may approach $infty$ at different rates, and this is the key point. Your professor's argument works only when $a$ and $b$ approach infinity equally fast, or in other words, when they are of equal order at infinity. In particular it is always valid if $a=b.$



                    So what about your professor's "proof"? Well, the ambiguity should now be obvious -- he uses the same symbol $infty$ for things that may behave differently. I think it's safe to assume he's thinking only of the variables are of equal order at infinity. Otherwise his proof breaks down since $infty-infty$ can be anything otherwise.



                    PS. However, you say an integral from $infty$ to $infty$ has no meaning to you. Well, I see you're thinking of ordering of the reals here. But note that we're not just dealing with the reals here, but the extended reals. As explained above, the best way to think of it is to think of the limits of the integral as approaching infinity at (not necessarily equal rates). Then it's easy to make sense of it. Another way may be to think of the one-point compactification of the real axis.






                    share|cite|improve this answer











                    $endgroup$



                    As has been pointed out by other answers, this is not always true because the symbol $infty$ can hide many things, even if we work with the extended reals. The actual meaning of the $infty$ is a limiting process as a certain variable becomes arbitrarily large. The upper and lower limits in the $$int_a^b f(x)mathrm d x$$ however may approach $infty$ at different rates, and this is the key point. Your professor's argument works only when $a$ and $b$ approach infinity equally fast, or in other words, when they are of equal order at infinity. In particular it is always valid if $a=b.$



                    So what about your professor's "proof"? Well, the ambiguity should now be obvious -- he uses the same symbol $infty$ for things that may behave differently. I think it's safe to assume he's thinking only of the variables are of equal order at infinity. Otherwise his proof breaks down since $infty-infty$ can be anything otherwise.



                    PS. However, you say an integral from $infty$ to $infty$ has no meaning to you. Well, I see you're thinking of ordering of the reals here. But note that we're not just dealing with the reals here, but the extended reals. As explained above, the best way to think of it is to think of the limits of the integral as approaching infinity at (not necessarily equal rates). Then it's easy to make sense of it. Another way may be to think of the one-point compactification of the real axis.







                    share|cite|improve this answer














                    share|cite|improve this answer



                    share|cite|improve this answer








                    edited 7 hours ago

























                    answered 7 hours ago









                    AllawonderAllawonder

                    2,9728 silver badges18 bronze badges




                    2,9728 silver badges18 bronze badges





















                        0












                        $begingroup$

                        As Peter Foreman mentioned, there are some occasions where you will get an integral of this form, however the exact integral does matter. Firstly visualise the error function:
                        $$operatornameerf(x)=frac2sqrtpiint_0^xe^-t^2dt$$
                        notice that:
                        $$lim_xto 0operatornameerf(x)=frac2sqrtpiint_0^0e^-t^2dt$$
                        Normally we can visualise an integral as area, but in this form it does not make sense since the range over which the area found $to0$. The most obvious case when this integral is zero, no matter the function, would be when both the bounds are equal as the range of the integral is then zero. Overall, it comes down to notation and making sure the bounds are properly defined.






                        share|cite|improve this answer









                        $endgroup$

















                          0












                          $begingroup$

                          As Peter Foreman mentioned, there are some occasions where you will get an integral of this form, however the exact integral does matter. Firstly visualise the error function:
                          $$operatornameerf(x)=frac2sqrtpiint_0^xe^-t^2dt$$
                          notice that:
                          $$lim_xto 0operatornameerf(x)=frac2sqrtpiint_0^0e^-t^2dt$$
                          Normally we can visualise an integral as area, but in this form it does not make sense since the range over which the area found $to0$. The most obvious case when this integral is zero, no matter the function, would be when both the bounds are equal as the range of the integral is then zero. Overall, it comes down to notation and making sure the bounds are properly defined.






                          share|cite|improve this answer









                          $endgroup$















                            0












                            0








                            0





                            $begingroup$

                            As Peter Foreman mentioned, there are some occasions where you will get an integral of this form, however the exact integral does matter. Firstly visualise the error function:
                            $$operatornameerf(x)=frac2sqrtpiint_0^xe^-t^2dt$$
                            notice that:
                            $$lim_xto 0operatornameerf(x)=frac2sqrtpiint_0^0e^-t^2dt$$
                            Normally we can visualise an integral as area, but in this form it does not make sense since the range over which the area found $to0$. The most obvious case when this integral is zero, no matter the function, would be when both the bounds are equal as the range of the integral is then zero. Overall, it comes down to notation and making sure the bounds are properly defined.






                            share|cite|improve this answer









                            $endgroup$



                            As Peter Foreman mentioned, there are some occasions where you will get an integral of this form, however the exact integral does matter. Firstly visualise the error function:
                            $$operatornameerf(x)=frac2sqrtpiint_0^xe^-t^2dt$$
                            notice that:
                            $$lim_xto 0operatornameerf(x)=frac2sqrtpiint_0^0e^-t^2dt$$
                            Normally we can visualise an integral as area, but in this form it does not make sense since the range over which the area found $to0$. The most obvious case when this integral is zero, no matter the function, would be when both the bounds are equal as the range of the integral is then zero. Overall, it comes down to notation and making sure the bounds are properly defined.







                            share|cite|improve this answer












                            share|cite|improve this answer



                            share|cite|improve this answer










                            answered 8 hours ago









                            Henry LeeHenry Lee

                            2,8421 gold badge4 silver badges19 bronze badges




                            2,8421 gold badge4 silver badges19 bronze badges



























                                draft saved

                                draft discarded
















































                                Thanks for contributing an answer to Mathematics Stack Exchange!


                                • Please be sure to answer the question. Provide details and share your research!

                                But avoid


                                • Asking for help, clarification, or responding to other answers.

                                • Making statements based on opinion; back them up with references or personal experience.

                                Use MathJax to format equations. MathJax reference.


                                To learn more, see our tips on writing great answers.




                                draft saved


                                draft discarded














                                StackExchange.ready(
                                function ()
                                StackExchange.openid.initPostLogin('.new-post-login', 'https%3a%2f%2fmath.stackexchange.com%2fquestions%2f3274013%2fintegral-from-infinity-to-infinity%23new-answer', 'question_page');

                                );

                                Post as a guest















                                Required, but never shown





















































                                Required, but never shown














                                Required, but never shown












                                Required, but never shown







                                Required, but never shown

































                                Required, but never shown














                                Required, but never shown












                                Required, but never shown







                                Required, but never shown







                                Popular posts from this blog

                                Invision Community Contents History See also References External links Navigation menuProprietaryinvisioncommunity.comIPS Community ForumsIPS Community Forumsthis blog entry"License Changes, IP.Board 3.4, and the Future""Interview -- Matt Mecham of Ibforums""CEO Invision Power Board, Matt Mecham Is a Liar, Thief!"IPB License Explanation 1.3, 1.3.1, 2.0, and 2.1ArchivedSecurity Fixes, Updates And Enhancements For IPB 1.3.1Archived"New Demo Accounts - Invision Power Services"the original"New Default Skin"the original"Invision Power Board 3.0.0 and Applications Released"the original"Archived copy"the original"Perpetual licenses being done away with""Release Notes - Invision Power Services""Introducing: IPS Community Suite 4!"Invision Community Release Notes

                                Canceling a color specificationRandomly assigning color to Graphics3D objects?Default color for Filling in Mathematica 9Coloring specific elements of sets with a prime modified order in an array plotHow to pick a color differing significantly from the colors already in a given color list?Detection of the text colorColor numbers based on their valueCan color schemes for use with ColorData include opacity specification?My dynamic color schemes

                                Tom Holland Mục lục Đầu đời và giáo dục | Sự nghiệp | Cuộc sống cá nhân | Phim tham gia | Giải thưởng và đề cử | Chú thích | Liên kết ngoài | Trình đơn chuyển hướngProfile“Person Details for Thomas Stanley Holland, "England and Wales Birth Registration Index, 1837-2008" — FamilySearch.org”"Meet Tom Holland... the 16-year-old star of The Impossible""Schoolboy actor Tom Holland finds himself in Oscar contention for role in tsunami drama"“Naomi Watts on the Prince William and Harry's reaction to her film about the late Princess Diana”lưu trữ"Holland and Pflueger Are West End's Two New 'Billy Elliots'""I'm so envious of my son, the movie star! British writer Dominic Holland's spent 20 years trying to crack Hollywood - but he's been beaten to it by a very unlikely rival"“Richard and Margaret Povey of Jersey, Channel Islands, UK: Information about Thomas Stanley Holland”"Tom Holland to play Billy Elliot""New Billy Elliot leaving the garage"Billy Elliot the Musical - Tom Holland - Billy"A Tale of four Billys: Tom Holland""The Feel Good Factor""Thames Christian College schoolboys join Myleene Klass for The Feelgood Factor""Government launches £600,000 arts bursaries pilot""BILLY's Chapman, Holland, Gardner & Jackson-Keen Visit Prime Minister""Elton John 'blown away' by Billy Elliot fifth birthday" (video with John's interview and fragments of Holland's performance)"First News interviews Arrietty's Tom Holland"“33rd Critics' Circle Film Awards winners”“National Board of Review Current Awards”Bản gốc"Ron Howard Whaling Tale 'In The Heart Of The Sea' Casts Tom Holland"“'Spider-Man' Finds Tom Holland to Star as New Web-Slinger”lưu trữ“Captain America: Civil War (2016)”“Film Review: ‘Captain America: Civil War’”lưu trữ“‘Captain America: Civil War’ review: Choose your own avenger”lưu trữ“The Lost City of Z reviews”“Sony Pictures and Marvel Studios Find Their 'Spider-Man' Star and Director”“‘Mary Magdalene’, ‘Current War’ & ‘Wind River’ Get 2017 Release Dates From Weinstein”“Lionsgate Unleashing Daisy Ridley & Tom Holland Starrer ‘Chaos Walking’ In Cannes”“PTA's 'Master' Leads Chicago Film Critics Nominations, UPDATED: Houston and Indiana Critics Nominations”“Nominaciones Goya 2013 Telecinco Cinema – ENG”“Jameson Empire Film Awards: Martin Freeman wins best actor for performance in The Hobbit”“34th Annual Young Artist Awards”Bản gốc“Teen Choice Awards 2016—Captain America: Civil War Leads Second Wave of Nominations”“BAFTA Film Award Nominations: ‘La La Land’ Leads Race”“Saturn Awards Nominations 2017: 'Rogue One,' 'Walking Dead' Lead”Tom HollandTom HollandTom HollandTom Hollandmedia.gettyimages.comWorldCat Identities300279794no20130442900000 0004 0355 42791085670554170004732cb16706349t(data)XX5557367